On variants of Chowla’s conjecture
Abstract.
We study the shifted convolution sums associated to completely multiplicative functions taking values in and give combinatorical proofs of two recent results in the direction of Chowla’s conjecture. We also determine the corresponding ”spectrum”.
Key words and phrases:
Chowla’s conjecture, Completely multiplicative functions, Shifted convolution sums, Spectrum2020 Mathematics Subject Classification:
Primary : 11N37, 11P32, Secondary : 11N35, 11T061. Introduction
1.1. Completely multiplicative functions
A function is called completely multiplicative if for every , . It follows that completely multiplicative functions are completely described by their values at primes. Let denote the Liouville function, defined to equal at the primes and extended to natural numbers completely multiplicatively. The study of various averages related to the Liouville function is an important part of analytic number theory and is intimately tied with many outstanding conjectures such as the Riemann hypothesis etc. One such question regards the -point correlations of the Liouville function, which is the focus of this paper. In particular we have the following conjecture of Chowla [3].
Conjecture 1 (Chowla).
Let denote the Liouville function and let be a non-empty set of non-negative integers, then
(1.1) |
There are many versions of this conjecture available in literature, a few replacing the Liouville function with the Möbius function (see [15] for more information and the inter-dependencies of various versions of Conjecture 1). In all the above variants, the conjecture encodes the expectation that the prime decomposition of an integer should be independent of that of for any fixed (or other suitable polynomial shifts). Thus the values of Liouville function at these integers maybe treated as “independent events” and hence their average should vanish in the long run (that is they should not be correlated). Extending this philosophy to -shifts gives Conjecture 1.
1.2. Main results
In this paper we give combinatorical proofs of two recent results regarding the shifted convolution averages of completely multiplicative functions taking values in , particularly along the lines of Conjecture 1. We begin with some notation. Let denote a subset of primes. Associated to , we define the completely multiplicative function as
(1.2) |
Equivalently, if denotes the completely additive function defined on the primes as
(1.3) |
then . Every completely multiplicative function taking values in is of the form for some . In fact this association to a completely multiplicative function taking values in , the set of primes where it takes the value is a group isomorphism where the subsets of primes are considered as a group under symmetric difference and the completely multiplicative functions are considered under pointwise multiplication.
Recall that a subset of natural numbers is called a small set if its sum of reciprocals converges and called a large set otherwise. For we define
(1.4) |
Define . Call a prime exceptional if it divides an element of and non-exceptional otherwise. We now have the first main theorem of this paper.
Theorem 1.
Let be a fixed subset of . Let be a small set of primes. Then, for every prime , there exists constants (depending on ) such that
(1.5) |
Moreover, for every non-exceptional prime, .
The above theorem can be deduced from the results in [10], but our proofs are different and comparitively elementary. What is interesting however is that the right hand side of (1.5) resembles an Euler product even when the corresponding arithmetic function namely fails to be even multiplicative. Moreover, it is not even clear that the associated Dirichlet series (namely ) continues beyond the half plane ; a property much needed for the application of Tauberian arguments. The multiplicative nature of the right hand side of (1.5) in turn, conforms with the principle that the “global” density should be a product of the “local” densities.
Theorem 1 still leaves open the question of large subsets of primes. In this case, we expect the limit on the left hand side of (1.5) to vanish, thus making Theorem 1 true in this case as well (if we interpret the infinite product on the right of (1.5) as zero, similar to the infinite product on the right hand side of (1.8) below). In particular, this would imply Conjecture 1. While such a generalization of Theorem 1 is out of reach, we are however able to prove the following.
Theorem 2.
If there exists such that , then . More concretely, if is a non constant completely multiplicative function, then for any ,
(1.6) |
and
(1.7) |
Theorem 2 follows from a recent work of Teravainen [20], which is much more general. As mentioned before the purpose of the present article is to give an elementary combinatorical proof.
Given a family of multiplicative functions taking values inside the unit circle, their “spectrum” was studied in [6]. Analogously, we define the spectrum as to be the topological closure of the set of all values as runs through all small subsets of primes for a fixed . The precise evaluation of below allows us to describe the spectrum below in §5.
1.3. Brief overview of the proof
We give a brief overview of the proof of Theorems 1 and Theorem 2 leaving the details for the following sections. Define and set (we shall show later on that in the cases that interest us, this natural density exists). The key idea in the proofs of Theorems 1 and 2 is the description of some structure in certain interesting families of subsets of .
For Theorem 1, the key ingredient is Lemma 2, which describes how behaves if we change and . This allows us to prove Theorem 1 for finite sets of primes and to let the set of primes grow “adding one prime at a time”. This leads to a limiting process which converges only if the original set is a small set. The relevant inequality is (3.9). As a consequence of the proof we describe how to calculate (see Example 1). From [10], is connected to the number of roots of the polynomial modulo powers of . We provide a slightly different description.
The proof of Theorem 2 involves a combinatorical argument which is rather independent in itself. The idea is to show that the collection of counterexamples satisfy some additional structure and symmetries (see Lemma 4). Using this we may reduce the the proof to the two-point correlation case, which is settled due to the work of Matomaki and Radziwill. The main arithmetic input, along with some handy lemmas, is Theorem 4 below (see also Remark 1).
1.4. Known results
Before we proceed with the proof, we mention some important results available in literature. The only case where Conjecture 1 is known is when , in which case the statement is classically equivalent to the prime number theorem. Using the fact that is completely multiplicative, we may derive (for )
where is the Riemann zeta function. Combining the works of Wintner [21] and Wirsing [22] leads to the following theorem (see also [1, Theorem 2]).
Theorem 3 (Wintner-Wirsing).
For any subset of the primes, we have
(1.8) |
An important aspect of the above theorem is the existence of the limit on the left hand side of (1.8).
Apart from the case of , it was not even known whether the limit in (1.1) exists. Even if it did exist, it was unknown whether there is sufficient cancellations to ensure that the limit is better than the trivial limit (that is ). This second question was settled only recently, due to the important work of Matomäki and Radziwiłł [11], where they studied the relation between “short” and “long” averages of arithmetic functions. We state, as a theorem, a corollary of the main result of [11].
Theorem 4 (Matomäki - Radziwiłł).
Let be an integer, then there exists a positive constant depending on and such that
for all large enough and for all non-empty. In other words,
(1.9) |
for any , and .
Most of the recent progress, particularly in the last decade or so regarding the Chowla’s conjecture (or more generally the Hardy-Littlewood-Chowla conjectures) have been from the perspective of ergodic theory. Following [11], there has been some progress towards Conjecture 1 particularly focusing on averaged [12, 13] and logarithmic [16, 18] versions. For conditional results assuming the existence of Siegel zeros, see [19]. For more information111The author wishes to thank Stelios Sachpazis for directing him to these references., see also [2, 9, 17, 4, 5, 14, 7].
Notation
-
(1)
Let denote the natural numbers and denote the non-negative integers .
-
(2)
The symbol will denote natural density (whenever it exists), and will denote the upper and lower natural densities respectively.
-
(3)
We shall denote the empty set as . The empty sum is defined to be zero and the empty product is defined to be one.
-
(4)
Given any two subsets , we define their “sum” as , and their “product” as . In particular, if , we also write and in place of and respectively.
-
(5)
The symbol will usually denote subsets of prime numbers unless otherwise stated. The symbol will usually refer to a finite subset of .
2. Groundwork
Given a sequence , we define the infinite product as . The product is considered to be convergent if and only if is convergent, in which case the limit above exists. If diverges to , then the product is defined to be . Given any two sets , their symmetric difference is defined as . In particular, contains elements of belonging to exactly one of or . Furthermore, by induction, the symmetric difference of a finite number of sets is precisely the collection of those elements belonging to exactly an odd number of the ’s. Suppose that is a non-empty subset of primes, we let denote the set of smooth integers. That is
It follows from our convention that for every and .
Lemma 1.
For any two subsets of the primes, and for any ,
Proof.
Suppose first that and are disjoint. Factorize as where is the largest divisor of in , that in and defined as . It follows that and . Thus it suffices to show that
(2.1) |
But observe that and likewise for the other two terms. Thus the lemma follows from the multiplicativity of the Liouville function.
For the general case write as the product where are respectively the largest divisors of in , and is defined as . Then on repeatedly applying (2.1), we have
∎
From our conventions, for any .
Lemma 2.
For any two subsets of the primes and two finite subsets of ,
Proof.
The proof is given in two parts. First we show that for any . If , the claim follows from definition, so suppose not. Let , where we have chosen . From Lemma 1 and complete multiplicativity,
Thus if and only if . But, we observe that and . Thus, if and only if . To conclude, if and only if , that is exactly one of them is . The latter condition is satisfied precisely on .
Now we show that . If either is the empty set, then the claim follows from definition, so we suppose not. Let and . Observe that if and only if is odd, and similarly for . Let .
If , then suppose without loss of generality that is odd for (say) (with ) and even otherwise. In particular exactly one of or is odd. Suppose without loss of generality that the former holds. Let denote the complete set of integers such that is odd. If is odd, then , and if is even, then . In any case . Therefore .
Now we prove the reverse inclusion. Suppose without loss of generality that . Let us suppose (with notation as above) that is even. The other case may be treated similarly. Choose as above. Then from our choices, it follows that is odd and is even. In particular, is odd and hence . This completes the proof. ∎
For , define the arithmetic progression
It follows from the Chinese remainder theorem that if are co-prime integers, then for any , there exists a unique such that
(2.2) |
Furthermore,
(2.3) |
for any . Combining (2.2) and (2.3), we get
whenever are co-prime. Define to be the collection all finite unions of the sets
Lemma 3.
For any non-empty subset of the primes, the following statements are true.
-
(1)
is closed under intersection.
-
(2)
is closed under complements.
-
(3)
Every element of can be written as a finite disjoint union of sets of the form for a fixed .
-
(4)
The natural density exists for every set .
Proof.
-
(1)
It is enough to prove this assertion for sets of the form for . If , we are done. Otherwise, is a finite union of sets of the form where is least common multiple of and . Therefore and we are done.
-
(2)
From above, it is sufficient to show that . Clearly .
-
(3)
Suppose . Denote by , the least common multiple of and set for every . We have,
The right hand side maybe written as a disjoint union by avoiding repetitions if there are any.
-
(4)
The natural density clearly exists for every set of the form and the claim follows from above. This completes the proof.
∎
Proposition 1.
Suppose that are non-empty mutually disjoint subsets of primes. Then for any and .
3. Proof of Theorem 1
3.1. Proof of Theorem 1 when is finite:
We simplify notation and write for , for etc.
Proposition 2.
Given a finite subset of the primes and and an integer , there exists subsets in such that
and
Proof.
The proof is by double induction, first on and then on . Let us first prove the statement for a fixed prime and when . Then contains precisely those integers such that an odd power of properly divides . In particular,
(3.1) |
Since the above union is a disjoint union, we have the inequalities,
(3.2) |
for any . Call the sets on the left hand side and right hand side of (3.2) as and respectively. We observe that . Letting , we see that exists and equals
(3.3) |
This completes the proof in this case.
Suppose that and consider . Let denote the natural projection map. From Lemma 2,
(3.4) |
In particular, from (3.3), it follows that
(3.5) |
But from (3.1), we may easily deduce that only if . Therefore we may rewrite the first equality in (3.4) as the disjoint union
(3.6) |
If is a non-constant function, then exists by induction on and we are done. In particular, we may suppose that or .
Suppose that is a constant function, say for some . If , then for any and hence . Therefore, only if . Moreover, in this situation,
Let . We observe that . Rewriting the above,
(3.7) |
In either case, it is enough to prove the proposition for . Now, if we consider the natural projection map , we may argue as above. If is a non-constant map, we are done. Otherwise we obtain (if necessary, after translation and scaling as above) , and the problem reduces to that of , with . Proceeding forward this process ultimately terminates because is strictly decreasing. In particular, there is a stage where the projection map (say ) is non-constant and we may then apply induction on . This completes the proof when . We note in passing that for large enough primes , is an injection and hence from (3.3) and (3.6) we have
Now fix and suppose that . We shall prove the proposition holds for supposing the same for and . From Lemma 2,
For every , choose from induction. Then, we have
From Lemma 3, both the left hand side and the right hand side above belong to . In particular the natural densities exist for each of those sets. Considering (upper) natural densities (and observing that ) we get,
From Proposition 1, we get
Similarly, we may also consider lower natural densities. Letting , we prove exists and equals
(3.8) |
∎
Example 1.
3.2. Proof of Theorem 1 when is a small set:
Suppose is fixed as above and that is an infinite (small) set and let . We show that exists and equals . It will be convenient to set . From Lemma 1, if and only if . Hence,
(3.9) |
where the last inequality follows from (3.5). As is a small set, the right hand side goes to zero as . This in particular shows exists because the left hand side of (3.9) dominates
Moreover, this also implies that thereby completing the proof.
Example 2.
Interestingly if and is a small set containing , the above calculations and proof give us that . More generally, for any given prime odd , we may also choose and any small set containing and get .
4. Proof of Theorem 2
We denote the complement of (in ) as . Recall that .
Lemma 4.
If there exists sets such that , then .
Proof.
We have
In particular, this means that on a set of natural density equal to . Reversing the roles of , we obtain that on a set of natural density equal to , and in particular on a set of natural density equal to . Thus we have shown that
completing the proof. ∎
We borrow the proof of the following lemma from combinatorics [8].
Lemma 5 (Wildon).
Suppose is a non-empty collection of finite subsets of which is closed under symmetric difference and translation, containing at least one non-empty set. Then contains a set of two elements.
By translation, we mean that if and is such that , then .
Proof.
For a given prime power , let denote the field of elements. Consider the map given by . By convention . We observe that and that . These two observations together tell us that is an ideal of . Suppose this ideal is generated by . Because is closed under translation as above, (equivalently ). Let be the splitting field of . Allowing for multiple roots if needed, we see that there exists an integer such that , that is, there exists a polynomial such that . If is so that , then
But the left hand side is an element of and hence . ∎
Proof of Theorem 2.
Suppose that is given such that for some (non-empty) . Let denote the collection of all finite subsets of such that . Clearly is a non-empty collection and is closed under translation. From Lemma 4, is closed under symmetric difference. Then by Lemma 5, contains a set of order two. This however contradicts Theorem 4 unless . This completes the proof. ∎
Remark 1.
Remark 2.
It is an desirable to prove an analogue of (1.9) in place of (1.6) and (1.7). However, the current proof does not seem to yield that strengthening. The challenge is to improve upon Lemma 4. More precisely, if two subsets of natural numbers have natural density equal to , then so does their intersection and this natural density is once again equal to . However, this is no longer true for upper or lower natural densities.
5. The spectrum of
Let be given. Define where the infimum runs over all the primes. Furthermore, we remark that there exists a prime such that .
Proposition 3.
With notation as above, .
Before we move on to the proof of Proposition 3, we require the following lemma. We give the proof for the sake of completeness.
Lemma 6.
For any given , and a parameter , there exists a small set of primes , with each prime larger than such that .
Proof.
We shall choose large enough without loss of generality and suppose that every prime is a non-exceptional prime. Choose a finite set such that . Such a choice is possible because the sum of reciprocals of primes is divergent. For simplicity, suppose . Choose disjoint from such that . Define . In the next step, we choose disjoint from and so on. Constructing ’s recursively as above, give for each
Multiplying out by gives us
(5.1) |
From the monotone convergence theorem, is convergent. But then (5.1) forces the limit to be . Now we choose to be . The fact that follows from Theorem 1 and the fact is a small set follows from the fact that . ∎
Proof of Proposition 3.
Choose so that for primes , . If , then the proposition is clear from Lemma 6. Otherwise choose , we shall show that there exists a such that . From Lemma 6, we may suppose that . Choose , and choose a small set of non-exceptional primes, each larger than , such that . Let be such that . Then from Theorem 1, the required small set of primes is . Thus .
Suppose . Suppose , it suffices to show that . This is clear from Theorem 1 since in this case for every prime , so that .
Suppose . For every small set , it then suffices to show that . Now observe that for every subset of the primes. Suppose for some fixed prime . For any small set , we have that . ∎
References
- [1] Peter Borwein, Stephen K. K. Choi, and Michael Coons. Completely multiplicative functions taking values in . Trans. Amer. Math. Soc., 362(12):6279–6291, 2010.
- [2] Jake Chinis. Siegel zeros and sarnak’s conjecture, 2021.
- [3] S. Chowla. The Riemann hypothesis and Hilbert’s tenth problem, volume Vol. 4 of Mathematics and its Applications. Gordon and Breach Science Publishers, New York-London-Paris, 1965.
- [4] Jean-Marie De Koninck, László Germán, and Imre Kátai. On the convolution of the Liouville function under the existence of Siegel zeros. Lith. Math. J., 55(3):331–342, 2015.
- [5] L. Germán and I. Kátai. On multiplicative functions on consecutive integers. Lith. Math. J., 50(1):43–53, 2010.
- [6] Andrew Granville and K. Soundararajan. The spectrum of multiplicative functions. Ann. of Math. (2), 153(2):407–470, 2001.
- [7] Harald Andrés Helfgott and Maksym Radziwiłł. Expansion, divisibility and parity, 2021.
- [8] Mark Wildon (https://mathoverflow.net/users/7709/mark wildon). Collection closed under symmetric difference and translation. MathOverflow. URL:https://mathoverflow.net/q/484822 (version: 2024-12-26).
- [9] Mikko Jaskari and Stelios Sachpazis. The chowla conjecture and landau-siegel zeroes, 2024.
- [10] Oleksiy Klurman. Correlations of multiplicative functions and applications. Compos. Math., 153(8):1622–1657, 2017.
- [11] Kaisa Matomäki and Maksym Radziwił ł. Multiplicative functions in short intervals. Ann. of Math. (2), 183(3):1015–1056, 2016.
- [12] Kaisa Matomäki, Maksym Radziwiłł, and Terence Tao. An averaged form of Chowla’s conjecture. Algebra Number Theory, 9(9):2167–2196, 2015.
- [13] Kaisa Matomäki, Maksym Radziwiłł, Terence Tao, Joni Teräväinen, and Tamar Ziegler. Higher uniformity of bounded multiplicative functions in short intervals on average. Ann. of Math. (2), 197(2):739–857, 2023.
- [14] Cédric Pilatte. Improved bounds for the two-point logarithmic chowla conjecture, 2023.
- [15] Olivier Ramaré. Chowla’s conjecture: from the Liouville function to the Moebius function. In Ergodic theory and dynamical systems in their interactions with arithmetics and combinatorics, volume 2213 of Lecture Notes in Math., pages 317–323. Springer, Cham, 2018.
- [16] Terence Tao. The logarithmically averaged Chowla and Elliott conjectures for two-point correlations. Forum Math. Pi, 4:e8, 36, 2016.
- [17] Terence Tao and Joni Teräväinen. Odd order cases of the logarithmically averaged Chowla conjecture. J. Théor. Nombres Bordeaux, 30(3):997–1015, 2018.
- [18] Terence Tao and Joni Teräväinen. The structure of logarithmically averaged correlations of multiplicative functions, with applications to the Chowla and Elliott conjectures. Duke Math. J., 168(11):1977–2027, 2019.
- [19] Terence Tao and Joni Teräväinen. The Hardy-Littlewood-Chowla conjecture in the presence of a Siegel zero. J. Lond. Math. Soc. (2), 106(4):3317–3378, 2022.
- [20] Joni Teräväinen. On the Liouville function at polynomial arguments. Amer. J. Math., 146(4):1115–1167, 2024.
- [21] Aurel Wintner. The Theory of Measure in Arithmetical Semi-Groups. publisher unknown, Baltimore, MD, 1944.
- [22] E. Wirsing. Das asymptotische Verhalten von Summen über multiplikative Funktionen. II. Acta Math. Acad. Sci. Hungar., 18:411–467, 1967.